Tough CR:

This topic has expert replies
Master | Next Rank: 500 Posts
Posts: 271
Joined: Tue May 22, 2012 3:22 am
Thanked: 7 times
Followed by:3 members

Tough CR:

by \'manpreet singh » Tue Aug 27, 2013 2:23 am
Politician: The bill that makes using car phones while driving illegal should be adopted. My support of this bill is motivated by a concern for public safety. Using a car phone seriously distracts the driver, which in turn poses a threat to safe driving. People would be deterred from using their car phones while driving if it were illegal to do so.
The argument's main conclusion follows logically if which one of the following is assumed?
(A) The more attention one pays to driving, the safer a driver one is.
(B) The only way to reduce the threat to public safety posed by car phones is through legislation.
(C) Some distractions interfere with one's ability to safely operate an automobile.
(D) Any proposed law that would reduce a threat to public safety should be adopted.
(E) Car phone use by passengers does not distract the driver of the car.

User avatar
Legendary Member
Posts: 643
Joined: Wed Aug 14, 2013 4:27 am
Thanked: 48 times
Followed by:7 members

by vinay1983 » Tue Aug 27, 2013 9:14 am
Tough one for me.I am stuck between A & B.

What is the source?
You can, for example never foretell what any one man will do, but you can say with precision what an average number will be up to!

Master | Next Rank: 500 Posts
Posts: 271
Joined: Tue May 22, 2012 3:22 am
Thanked: 7 times
Followed by:3 members

by \'manpreet singh » Wed Aug 28, 2013 8:09 am
The question is actually quite tough mate, it another one from kaplan mock.

And the OA is A

Since the politician assumes if the distraction is removed than one would be more attentive while driving and hence more safe his drive would be.If you negate this whole argument while fall apart.

Option B is also a extreme option(Usage of only way)In gmat such extreme options are seldom correct.

User avatar
Legendary Member
Posts: 643
Joined: Wed Aug 14, 2013 4:27 am
Thanked: 48 times
Followed by:7 members

by vinay1983 » Wed Aug 28, 2013 8:41 am
Missed it, I usually observe such words and phrases.

I am still confused though, on second thoughts A and D appear good enough.maybe Kaplan people might pitch in or THE EXPERTS are needed here!
You can, for example never foretell what any one man will do, but you can say with precision what an average number will be up to!

GMAT/MBA Expert

User avatar
Elite Legendary Member
Posts: 10392
Joined: Sun Jun 23, 2013 6:38 pm
Location: Palo Alto, CA
Thanked: 2867 times
Followed by:511 members
GMAT Score:800

by [email protected] » Wed Aug 28, 2013 1:18 pm
Hi All,

This CR question uses a version of "causality", but in a way that you might not be used to thinking about it.

Causality is the idea that one thing will cause another.

For example, If you study, then your performance will improve. The causal argument is that studying ---> improved performance

Here, the author is concerned with "public safety". He claims that IF you're driving a car and IF you're on your cell phone, THEN you will be seriously distracted AND a threat to safe driving (which is an implied threat to public safety). So, this causality has a lot of pieces to it.

Since the author's goal is public safety, and he's only talking about the above details, he has to assume that IF you're driving AND IF you don't use a cell phone while driving, THEN you won't be as distracted (and thus less of a threat to public safety).

The correct answer doesn't list out every part of the causality, it only lists out part of it (which is still correct, by the way): Less distracted ---> safer driver.

GMAT assassins aren't born, they're made,
Rich
Contact Rich at [email protected]
Image

User avatar
Master | Next Rank: 500 Posts
Posts: 184
Joined: Sun Aug 19, 2012 10:04 pm
Thanked: 10 times
Followed by:2 members

by Mission2012 » Tue Sep 03, 2013 9:30 am
Rich,

I am bit confused.
What is the main conclusion of this argument.

I thought that main conclusion is -

People would be deterred from using their car phones while driving if it were illegal to do so.


'manpreet singh wrote:Politician: The bill that makes using car phones while driving illegal should be adopted. My support of this bill is motivated by a concern for public safety. Using a car phone seriously distracts the driver, which in turn poses a threat to safe driving. People would be deterred from using their car phones while driving if it were illegal to do so.
The argument's main conclusion follows logically if which one of the following is assumed?
(A) The more attention one pays to driving, the safer a driver one is.
(B) The only way to reduce the threat to public safety posed by car phones is through legislation.
(C) Some distractions interfere with one's ability to safely operate an automobile.
(D) Any proposed law that would reduce a threat to public safety should be adopted.
(E) Car phone use by passengers does not distract the driver of the car.
If you find my post useful -> please click on "Thanks"

User avatar
Master | Next Rank: 500 Posts
Posts: 234
Joined: Tue Jul 16, 2013 9:00 am
Location: West Virginia
Thanked: 9 times

by Java_85 » Tue Sep 03, 2013 4:11 pm
I'm between A and D, and think both can be the right answer, why D is not right?

User avatar
GMAT Instructor
Posts: 2193
Joined: Mon Feb 22, 2010 6:30 pm
Location: Vermont and Boston, MA
Thanked: 1186 times
Followed by:512 members
GMAT Score:770

by David@VeritasPrep » Tue Sep 03, 2013 6:16 pm
STOP! Put on the brakes. This is not a Kaplan question and the answer is not A

OA D

This an LSAT question from June 1997. It is question 13 of the 2nd Logical Reasoning section of that particular LSAT. Furthermore, this is a type of question that DOES NOT APPEAR on the GMAT.

This question is one where you are asked to GUARANTEE the conclusion. That is what is meant by "follows logically." You are not only required to support the conclusion you are actually required to make it a totally valid conclusion.

The conclusion to this question is "The bill that makes using car phones while driving illegal should be adopted." You can always find the correct conclusion by using the "WHY Test?" If you ask "WHY?" about the conclusion the rest of the arguments should answer.

Ask "why should we adopt this bill to make using car phones illegal?" See that all of the premises provide the answers? That is how we know that it is the proper conclusion.

So you need to have an answer choice that makes this conclusion absolutely true. Now if this were the LSAT I would teach you to do this question this way. Look for the extent of the evidence and then see what needs to be added to get you from the evidence to the conclusion.

Now the evidence tells you that using a cell phone distracts the driver which is a threat to public safety. And the evidence also indicates that a law would be effective in reducing use of cell phones. So we can say that the law would prevent a threat to public safety.

The conclusion is that the law should be adopted.

If you insert choice D you see that any law that reduces a threat to public safety should be adopted. So that takes you from the evidence to the conclusion and this guarantees the conclusion.

Again this is not a type of question that you would normally see on the GMAT.

Since this is an LSAT question, you might want to check out this link to an article I wrote on using the LSAT questions to study for the GMAT. https://www.beatthegmat.com/lsat-to-stud ... 69915.html
Veritas Prep | GMAT Instructor

Veritas Prep Reviews
Save $100 off any live Veritas Prep GMAT Course

User avatar
GMAT Instructor
Posts: 578
Joined: Tue Aug 25, 2009 6:00 pm
Thanked: 136 times
Followed by:62 members

by KapTeacherEli » Thu Sep 05, 2013 5:41 pm
David@VeritasPrep wrote:STOP! Put on the brakes. This is not a Kaplan question
Thanks for catching this :-) I was trying to find this problem in our tests to reference the explanation, and couldn't--this explains why!
Eli Meyer
Kaplan GMAT Teacher
Cambridge, MA
www.kaptest.com/gmat

ImageImageImage

Senior | Next Rank: 100 Posts
Posts: 30
Joined: Sat Aug 24, 2013 10:46 am

by Nitin.811g » Sat Sep 07, 2013 3:11 am
David@VeritasPrep wrote:STOP! Put on the brakes. This is not a Kaplan question and the answer is not A

OA D

This an LSAT question from June 1997. It is question 13 of the 2nd Logical Reasoning section of that particular LSAT. Furthermore, this is a type of question that DOES NOT APPEAR on the GMAT.
Thanks David for the explanation.

As you mentioned that this is not really a GMAT question, gives me a sigh of relief.

But please help me understand this. If this question were to appear on GMAT, would we still go with D because strong keywords like Any, Only, All are usually not correct in GMAT answers.

Please suggest.

User avatar
GMAT Instructor
Posts: 2193
Joined: Mon Feb 22, 2010 6:30 pm
Location: Vermont and Boston, MA
Thanked: 1186 times
Followed by:512 members
GMAT Score:770

by David@VeritasPrep » Sat Sep 07, 2013 4:55 am
Nitin.811g

What you have said is partially correct. Strong or absolute words like "All" "None" Must" "Only" and in this question "Any" are not usually preferred in Inference questions and Assumption questions. These types of questions usually take a more flexible word like "might" "some" or "may."

HOWEVER, Strengthen and Weaken questions, (and this is an extreme strengthen question and not an assumption) do usually prefer the strong answer choices. So D would always be the correct answer to this question.

Now, if you so any more LSAT questions you will need to know this trick. On the GMAT the word assume or assumption is a give away that you have an assumption question. But not on the LSAT. There are strengthen questions that use the word "assume".

Here is the test - if you can replace the word "assume" with the word "true" then you can safely IGNORE the "assume" and it is not an assumption question.

For example, this question stem says "The argument's main conclusion follows logically if which one of the following is assumed?" Replace the assumed with "true". "The argument's main conclusion follows logically if which one of the following is TRUE?

It does work, so you can ignore the assumed and this is a strengthen question. It is actually, as I noted above an extreme strengthen question where you literally have to prove the conclusion true. Something not found on the GMAT.

The moral of the story is know where your questions come from! And if you are going to use LSAT questions be careful!

(and read my article above about using LSAT questions to study for the GMAT). By the way, since I wrote that article there are more GMAT-specific questions available that you should use instead of resorting to LSAT questions. The GMATPrep Pack add-on is available with many CR questions not found elsewhere and there is the new Free Veritas Question Bank at https://www.veritasprep.com/gmat-question-bank/ and the free Veritas practice test at https://www.veritasprep.com/gmat/free-gm ... tice-test/

I am not saying don't use the LSAT questions is you are out of questions and need some more, just be sure you know what you are doing!
Veritas Prep | GMAT Instructor

Veritas Prep Reviews
Save $100 off any live Veritas Prep GMAT Course

Senior | Next Rank: 100 Posts
Posts: 30
Joined: Sat Aug 24, 2013 10:46 am

by Nitin.811g » Sat Sep 07, 2013 10:29 am
Thanks David, that really helps...